alich100

New Member
ارسال ها
202
لایک ها
90
امتیاز
0
#41
پاسخ : مسابقه ریاضی ((حل کنید !))

احتمالا منظورتون این بوده:

درسته؟
 

Yousefi

Well-Known Member
ارسال ها
432
لایک ها
602
امتیاز
93
#42
پاسخ : مسابقه ریاضی ((حل کنید !))

ببخشید :9: بازم یه اشتباه تایپیه دیگه

این سوال دیگه درسته درسته ، غلط هم نداره !

می دانیم که :

و
دنباله ای از اعداد حقیقی است.

ثابت کنید که :



احتمالا منظورتون این بوده:

درسته؟
بله
 
آخرین ویرایش توسط مدیر

happymoj

Active Member
ارسال ها
346
لایک ها
150
امتیاز
43
#43
پاسخ : مسابقه ریاضی ((حل کنید !))

طرفین رو به توان برسونید با استفاده از نامساوی جلو آسون اثبات می شه

اینم چون برای اعداد طبیعیه یه استقرای سادس
 

Yousefi

Well-Known Member
ارسال ها
432
لایک ها
602
امتیاز
93
#44
پاسخ : مسابقه ریاضی ((حل کنید !))

طرفین رو به توان برسونید با استفاده از نامساوی جلو آسون اثبات می شه

اینم چون برای اعداد طبیعیه یه استقرای سادس
دنبال یه روش بهترم :57:
 

hkh74

New Member
ارسال ها
213
لایک ها
392
امتیاز
0
#45
پاسخ : مسابقه ریاضی ((حل کنید !))

می دانیم که :

و
دنباله ای از اعداد حقیقی است.

ثابت کنید که :


دنبال یه روش بهترم

تساوی اول و نامساوی آخر(مربعی-حسابی) واضحند.

برای اثبات اولین نامساوی: (اثبات هندسی)

مستطیل های
را با رأس های
(i=1,2,...,n) در نظر بگیرید. مساحت مستطیل k ام برابر است با
. پس مجموع مساحت های این مستطیل های دو به دو جدا از هم برابر است با طرف چپ نامساوی.

مستطیل های
را با رأس های
(i=1,2,...,n) در نظر بگیرید. مساحت مستطیل k ام برابر است با
. پس مجموع مساحت های این مستطیل های دو به دو جدا از هم برابر است با طرف راست نامساوی.


حالا کافیه ثابت کنید مستطیل های
مستطیل های
رو میپوشونن... که اگر شکلش رو بکشید ایده ی اثبات این قسمت خیلی راحت به ذهن میرسه.
 

Yousefi

Well-Known Member
ارسال ها
432
لایک ها
602
امتیاز
93
#46
پاسخ : مسابقه ریاضی ((حل کنید !))

بسیار عالی بود، حالا من راه خودمو میگذارم : ( خودم که نه ! )

یه جورایی شبیه روش تغییر متغیره !

و حالا کافیه از دو طرف جذر بگیریم.

حالا یکی باید یه سوال بگذاره ، در ضمن به جناب hkh74 و alich100 هم امتیاز اضافه میشه.
 

POURIYA- F

New Member
ارسال ها
107
لایک ها
53
امتیاز
0
#47
پاسخ : مسابقه ریاضی ((حل کنید !))

آیا تابع کران دار
وجود دارد به طوری که :





* توجه کنید که
.
 
آخرین ویرایش توسط مدیر

mehran88

New Member
ارسال ها
640
لایک ها
1,232
امتیاز
0
#48

alich100

New Member
ارسال ها
202
لایک ها
90
امتیاز
0
#49
پاسخ : مسابقه ریاضی ((حل کنید !))

مستر/میس hkh74 آخرین نامساویتون اشتباهه
یعنی شما می گید:

قرار دهید:

داریم:

!!!
تازه فک کنم همیشه این نامساوی برعکس باشه!!!
درسته؟
 

alich100

New Member
ارسال ها
202
لایک ها
90
امتیاز
0
#50
پاسخ : مسابقه ریاضی ((حل کنید !))

منظورتون از
توانه یا
؟
 

zz_torna2

New Member
ارسال ها
300
لایک ها
254
امتیاز
0
#51
پاسخ : مسابقه ریاضی ((حل کنید !))

مستر/میس hkh74 آخرین نامساویتون اشتباهه
یعنی شما می گید:

قرار دهید:

داریم:

!!!
تازه فک کنم همیشه این نامساوی برعکس باشه!!!
درسته؟
راست میگیا حواسم نبود اثبات گفته تون:


حالا نامساوی ینسن میزنیم:

 

zz_torna2

New Member
ارسال ها
300
لایک ها
254
امتیاز
0
#52
پاسخ : مسابقه ریاضی ((حل کنید !))

اصلا لازم نیست ینسن بزنیم .با نامساوی مربع-حسابی هم اثبات میشه!!:4:
 

POURIYA- F

New Member
ارسال ها
107
لایک ها
53
امتیاز
0
#53

POURIYA- F

New Member
ارسال ها
107
لایک ها
53
امتیاز
0
#55

zz_torna2

New Member
ارسال ها
300
لایک ها
254
امتیاز
0
#56
پاسخ : مسابقه ریاضی ((حل کنید !))

سوال و جواب:
ثابت کنید:


راه حل:

(اثبات هندسی)

مستطیل های
را با رأس های
(i=1,2,...,n) در نظر بگیرید. مساحت مستطیل k ام برابر است با
. پس مجموع مساحت های این مستطیل های دو به دو جدا از هم برابر است با طرف چپ نامساوی.

مستطیل های
را با رأس های
(i=1,2,...,n) در نظر بگیرید. مساحت مستطیل k ام برابر است با
. پس مجموع مساحت های این مستطیل های دو به دو جدا از هم برابر است با طرف راست نامساوی.


حالا کافیه ثابت کنید مستطیل های
مستطیل های
رو میپوشونن... که اگر شکلش رو بکشید ایده ی اثبات این قسمت خیلی راحت به ذهن میرسه.
 

alich100

New Member
ارسال ها
202
لایک ها
90
امتیاز
0
#57
پاسخ : مسابقه ریاضی ((حل کنید !))

اثبات سوال
به یه شکل دیگه(البته به زیبایی اثبات hkh74 نمی رسه!)

طبق نامساوی چبیشف (متشابه الترتیب) داریم:


بهتره دیگه روی سوال آقای پوریا بفکریم!
 
آخرین ویرایش توسط مدیر

mahanmath

New Member
ارسال ها
898
لایک ها
701
امتیاز
0
#58
پاسخ : مسابقه ریاضی ((حل کنید !))


تساوی اول و نامساوی آخر(مربعی-حسابی) واضحند.

برای اثبات اولین نامساوی: (اثبات هندسی)

مستطیل های
را با رأس های
(i=1,2,...,n) در نظر بگیرید. مساحت مستطیل k ام برابر است با
. پس مجموع مساحت های این مستطیل های دو به دو جدا از هم برابر است با طرف چپ نامساوی.

مستطیل های
را با رأس های
(i=1,2,...,n) در نظر بگیرید. مساحت مستطیل k ام برابر است با
. پس مجموع مساحت های این مستطیل های دو به دو جدا از هم برابر است با طرف راست نامساوی.


حالا کافیه ثابت کنید مستطیل های
مستطیل های
رو میپوشونن... که اگر شکلش رو بکشید ایده ی اثبات این قسمت خیلی راحت به ذهن میرسه.
با این ایده معروف این مسئله رو هم حل کنید :
AoPS Forum - Problem 5, Iberoamerican Olympiad 2011 • Art of Problem Solving

یه مسئله تورنمت هم هست که اینطوری حل میشه ولی دقیق صورتش یادم نیست :71:
 

Yousefi

Well-Known Member
ارسال ها
432
لایک ها
602
امتیاز
93
#59
پاسخ : مسابقه ریاضی ((حل کنید !))

بسیار عالی بود، حالا من راه خودمو میگذارم : ( خودم که نه ! )

یه جورایی شبیه روش تغییر متغیره !

و حالا کافیه از دو طرف جذر بگیریم.

حالا یکی باید یه سوال بگذاره ، در ضمن به جناب hkh74 و alich100 هم امتیاز اضافه میشه.
چند تا سوال !؟
من که اثباتش کردم دیگه چرا این همه پست می زنید ؟
چرا zz_torna2 بجای hkh74 پست زدند ؟

راست میگیا حواسم نبود اثبات گفته تون:
چرا POURIYA_F به آقای شریفی جواب دادن ؟؟؟ ایشون که اصلا پست نزدن!
 

mehran88

New Member
ارسال ها
640
لایک ها
1,232
امتیاز
0
#60
پاسخ : مسابقه ریاضی ((حل کنید !))


چند تا سوال !؟
من که اثباتش کردم دیگه چرا این همه پست می زنید ؟
چرا zz_torna2 بجای hkh74 پست زدند ؟


چرا POURIYA_F به آقای شریفی جواب دادن ؟؟؟ ایشون که اصلا پست نزدن!
آقای یوسفی1
بهت قول میدم کارآگاه خوبی خواهی شد! سوالات خوبی میپرسی برای رسیدن به سرنخ!

سوال 1- فکر کنم جناب تورنا می خواستند سوال و جواب رو یه دفعه توی یه پست بذارن که بچه ها استفاده کنند.


سوال 2- اگه دقت کرده باشی آقای شریفی پست پوریا خان رو ویرایش کرده بودند که اقا پوریا هم یه پست خطاب به استاد زدند!


ولی من پلیس بهتری میشم قطعا!!
 
بالا